Torts Adaptibar

Ace your homework & exams now with Quizwiz!

For five years, a rancher had kept his horse in a ten-acre field enclosed by a six-foot woven wire fence with six inches of barbed wire on top. The gate to the field was latched and could not be opened by an animal. The rancher had never had any trouble with people coming onto his property and bothering the horse, and the horse had never escaped from the field. One day, however, when the rancher went to the field, he found that the gate was open and the horse was gone. Shortly before the rancher

A or C is correct. Although rare, on a few occasions the NCBE has released two correct answers for one question. For this question, when it was scored, either answer A or C was accepted as correct and given credit. However, moving forward, examinees should still approach each question as if only one answer is correct.

A defendant, an inexperienced driver, borrowed a car from the plaintiff, a casual acquaintance, for the express purpose of driving it several blocks to the local drug store. Instead, the defendant drove the car, which then was worth $12,000, 100 miles to another city. While the defendant was driving in the other city the next day, the car was hit by a negligently driven truck and sustained damage that will cost $3,000 to repair. If repaired, the car will be fully restored to its former condition

A: $12,000. A is correct. The plaintiff should recover the fair market value of the car, $12,000, because the defendant substantially interfered with the plaintiff's chattel by driving the car 100 miles, keeping it overnight, and incurring $3,000 in damages. This amounts to a conversion claim, which allows for damages for the full value of the car at the time of the conversion.

A school bus driver reported to a middle school principal that a student had harassed other children on the bus. The principal informed the student's parents of the bus driver's report and told them that, because of the student's behavior, the student could not ride the bus for the next week and would have to be driven to school by a parent. The following Monday morning, after the bus driver had let the children off the bus in front of the school, but before she could close the door and dr

A: Assault. A is correct. Assault would be the best theory under which the driver could bring a claim against the father. A reasonable jury could conclude that the father intentionally caused the driver to apprehend imminent harmful physical contact, as is required for assault liability.

A patient received anesthesia while giving birth. Upon awakening from the anesthesia, she discovered a severe burn on the inner portion of her right knee. The patient has brought a medical malpractice action in which she has joined all of the physicians and nurses who exercised control over her person, the delivery room, the medical procedures, and the equipment used during the period in which she was unconscious. The defendants have jointly moved for summary judgment. The patient has produced a

A: At least one of the defendants had control over whatever agency or instrumentality caused the patient's injury. A is correct. Showing that a defendant had control over the instrumentality that caused her injury would be a basis for the application of res ipsa loquitur in some jurisdictions and, if res ipsa loquitur was applied, it could be a basis for defeating the defendants' motion for summary judgment.

A landowner hired a tree specialist to cut down four trees, which he pointed out to the specialist before the specialist began work. Although the landowner reasonably believed that all the trees were on his property, three of the trees that were cut down were in fact on a neighbor's property. Who, if anyone, is liable to the landowner's neighbor for conversion? A: Both the tree specialist and the landowner. B: Neither the tree specialist nor the landowner. C: The landowner only. D: The

A: Both the tree specialist and the landowner. A is correct. Conversion occurs when an actor intentionally interacts with an item that is the personal property of another so as to permanently deprive the rightful owner of possession. It is irrelevant whether the actor knows that the item belongs to another individual. Thus, the landowner's reasonable belief that the trees were his does not defeat liability for either the landowner or the tree specialist.

A plaintiff owned a large tract of land on the shore of a lake. The defendant lived on a stream that ran along one boundary of the plaintiff's land and into the lake. At some time in the past, a channel had been cut across the plaintiff's land from the stream to the lake at a point some distance from the mouth of the stream. From where the defendant lived, the channel served as a convenient shortcut to the lake. Erroneously believing that the channel was a public waterway, the defendant made fre

A: Judgment for the plaintiff for nominal damages, because the defendant intentionally used the channel. A is correct. Trespass is an intentional entry onto the land of another, without permission. The intent only refers to the intent to enter the property; the defendant need not know that it is another's private property. For an intentional trespass to land, damage is not required; the court will award nominal damages based on the trespass alone. The facts here support a claim for trespass because the defendant intentionally used the channel, without permission, even though he incorrectly believed it was a public waterway. All other answer choices may be eliminated because they find in favor of the defendant.

A construction company was digging a trench for a new sewer line in a street in a high-crime neighborhood. During the course of the construction, there had been many thefts of tools and equipment from the construction area. One night, the construction company's employees neglected to place warning lights around the trench. A delivery truck drove into the trench and broke an axle. While the truck driver was looking for a telephone to call a tow truck, thieves broke into the truck and stole $350

A: Deny both motions, because there is evidence to support a finding that the construction company should have realized that its negligence could create an opportunity for a third party to commit a crime. A is correct. A defendant will be liable for the criminal actions of a third party if they were a reasonably foreseeable result at the time of the defendant's negligence. Here, the construction company claims it should not be liable for third party criminal acts, and yet, it had knowledge of regular thefts in the area, which means that a jury could reasonably decide that it was foreseeable. Because of this disputed issue, the motions should both be denied and it should be submitted to the jury.

A longshoreman fell to his death through an open hatch on the deck of a ship. The longshoreman was an employee of a company that had contracted with the ship's owner to load and unload the ship. The fall occurred at night, when loading work was over for the day, and there was no reason for the longshoreman to have been near the hatch. A negligence action was filed against the ship's owner for the death of the longshoreman. In that action, the owner has moved for summary judgment and has prov

A: Deny the motion and submit the case to the jury with instructions that the custom is relevant but not conclusive on the issue of negligence. A is correct. The motion should be denied because, although it is custom for the hatches to be left open for ventilation, it cannot be said to be conclusive on the issue of negligence. There are not enough facts in this question to determine if that alone absolves the ship's owner of liability. B is incorrect. Even if the longshoreman was impro

After a car buyer failed to make timely payments on her auto loan and failed to respond to notices of default properly sent to her home address, the loan company hired a collection agency to repossess the car. An employee of the agency went to the buyer's home and knocked on the front door. When the buyer answered, the employee explained that he was there to repossess the car and asked for the car keys. The buyer handed the employee the keys but then asked the employee to allow her to retrieve

A: No, because a fact-finder could reasonably conclude that the buyer used excessive force in attempting to defend her property. A is correct. Summary judgment will be granted if a party shows there is no genuine dispute of material fact and that the party is entitled to judgment as a matter of law. Fed. R. Civ. P. 56. The buyer intentionally shoved the employee, which satisfies the prima facie case for battery. As an affirmative defense, the buyer could claim that she was privileged to use force against the employee in order to maintain possession of her property. However, this defense will be established only if the jury reasonably concludes that the buyer's use of force was reasonable, not excessive, under the circumstances. Thus, the trial court should deny the buyer's motion for summary judgment because there is a dispute as to whether the buyer's use of force was reasonable.

As a bartender was removing the restraining wire from a bottle of champagne produced and bottled by Winery, Inc., the plastic stopper suddenly shot out of the bottle. The stopper struck and injured the bartender's eye. The bartender had opened other bottles of champagne, and occasionally the stoppers had shot out with great force, but the bartender had not been injured. The bartender has brought an action against Winery, Inc., alleging that the bottle that caused his injury was defective and unr

A: No, because a legally sufficient warning would not have prevented the bartender's injury. A is correct. This is a products liability action based on strict liability for a design defect, specifically an inadequate warning. In order to establish a prima facie case in products liability based on strict liability, the following elements must be proven: (i) the defendant is a commercial supplier; (ii) the defendant produced or sold a defective product; (iii) the defective product was the actual and proximate cause of the plaintiff's injury; and (iv) the plaintiff suffered damages to person or property. In order to prove a defective product, courts use a factor balancing test known as the feasible alternative approach. One factor is the avoidability of the injury by care in use, including the role of instructions and warnings. In this case, the bartender was aware of the risk and had experienced it before. A legally sufficient warning here would not have prevented the bartender's inj

A defendant has a small trampoline in his backyard which, as he knows, is commonly used by neighbor children as well as his own. The trampoline is in good condition, is not defective in any way, and normally is surrounded by mats to prevent injury if a user should fall off. Prior to leaving with his family for the day, the defendant leaned the trampoline up against the side of the house and placed the mats in the garage. While the defendant and his family were away, the plaintiff, aged 11, a new

A: No, because children likely to be attracted by the trampoline would normally realize the risk of using it without mats. A is correct. Here, the facts indicate that while the neighborhood children commonly used the defendant's trampoline, the plaintiff was new to the neighborhood and so was unknown to the defendant. The plaintiff is also 11, old enough to understand the risks. The defendant knew that the local children would be aware of the need for mats because those children "commonly" used the trampoline and the mats. Therefore, the plaintiff will not prevail.

A bus passenger was seated next to a woman whom he did not know. The woman stood to exit the bus, leaving a package on the seat. The passenger lightly tapped the woman on the back to get her attention and to inform her that she had forgotten the package. Because the woman had recently had back surgery, the tap was painful and caused her to twist and seriously injure her back. If the woman sues the passenger to recover for the back injury, will she be likely to prevail? A: No, because she is pre

A: No, because she is presumed to have consented to the ordinary contacts of daily life. A is correct. The woman gave no indication that she did not want to be subjected to the ordinary touches that are part of life in a crowded society. In the absence of such an indication from her, the passenger was entitled to believe that she implicitly consented to a light tap to get her attention. The passenger's touch was neither unreasonable nor inconsistent with ordinary social norms privileging such contacts, and would not amount to offensive or harmful contact sufficient to give rise to a claim for battery.

During a deer-hunting season open to rifle hunters, a hunter saw a deer in the forest. He shot his rifle at the deer, hoping to hit and kill it. Instead, he hit and injured a hiker. The hunter had not realized that the hiker was there. Does the injured hiker have an actionable battery claim against the hunter? A: No, because the hunter did not intend to shoot the hiker. B: No, because the hunter did not make direct physical contact with the hiker. C: Yes, because the bullet from the hunter'

A: No, because the hunter did not intend to shoot the hiker. A is correct. The hunter intended to kill and shoot the deer, not to hit the hiker. Th

A food company contracted with a delivery service to supply food to remote areas around the world. The contract between the food company and the delivery service was terminable at will. The delivery service then entered into a contract with an airline to provide an airplane to deliver the food. The contract between the delivery service and the airline was also terminable at will. The food company was displeased with the airline because of a previous business dispute between them. Upon learning o

A: No, because the airline and the delivery service were the parties to the contract. A is correct. The airline company sued the delivery service based on a theory of the tortious interference with its contract. The tort of interference with contract provides a cause of action against those who improperly interfere with the performance of a contract between the plaintiff and a third person. In this case, the airline and the delivery service were parties to a contract, and any action between them would be based on the contract, rather than on tort. The proper defendant in the tort action would be the food company. See Restatement (Second) of Torts § 766.

An 11-year-old boy was driving a full-size motorcycle on a private road, where the boy was a trespasser. The motorcycle hit a tire that had fallen off a truck driven by a delivery company employee who was making a delivery to an address on the private road. The boy was injured when his motorcycle went out of control after striking the tire. In a negligence action brought on behalf of the boy against the delivery company, the company contends that the boy was contributorily negligent and that his

A: No, because the boy was driving a motorcycle. A is correct. Children engaging in a dangerous activity that is characteristically undertaken by adults may be held to an adult standard of care. Because the boy was driving a motorcycle—an adult activity—he will be held to the adult standard of care.

A man tied his dog to a bike rack in front of a store and left the dog there while he went inside to shop. The dog was usually friendly and placid. A five-year-old child started to tease the dog by pulling gently on its ears and tail. When the man emerged from the store and saw what the child was doing to the dog, he became extremely upset. Does the man have a viable claim against the child for trespass to chattels? A: No, because the child did not injure the dog. B: No, because the child was

A: No, because the child did not injure the dog. A is correct. Without any actual harm to the dog or any substantial deprivation of the man's use of his dog, a trespass to chattels claim will not succeed.

In a tavern, an intoxicated woman threatened to slash a man with a broken beer bottle. Another customer, who had not been threatened by the woman, forcefully grabbed the woman and locked her in the tavern's storeroom until the police could arrive. In the process, although the customer used reasonable force, the customer badly sprained the woman's wrist. Is the woman likely to recover in an action against the customer? A: No, because the customer's conduct was privileged as a defense of o

A: No, because the customer's conduct was privileged as a defense of others. A is correct. A defense based on the defense of others is available when the defendant has a reasonable belief that the person being aided would have the right of self-defense. The defendant may use as much force as he could have used in self-defense if the injury were threatened to him. Here, an intoxicated woman threatened to slash a man with a broken beer bottle. The man would have the right of self-defense and therefore the woman will not recover in an action against the customer.

A shopper was riding on an escalator in a department store when the escalator stopped abruptly. The shopper lost her balance and fell down the escalator steps, sustaining injuries. Although the escalator had been regularly maintained by an independent contractor, the store's obligation to provide safe conditions for its invitees was nondelegable. The shopper has brought an action against the store for damages, and the above facts are the only facts in evidence. The store has moved for a directed

A: No, because the finder of fact could infer that the escalator malfunction was due to negligence. A is correct. There is enough evidence here to support an inference of negligence on the part of the store or the contractor. A jury could find that the malfunction was due to the negligent installation, maintenance, or operation of the escalator; the store would be responsible for all these possible causes under the nondelegable duty doctrine.

A man rented a car from a car rental agency. Unbeknownst to the rental agency, the car had a bomb hidden in it at the time of the rental. The bomb exploded an hour later, injuring the man. Immediately prior to renting the car to the man, the rental agency had carefully inspected the car to be sure it was in sound operating condition. The rental agency did not inspect for hidden explosive devices, but such an inspection would have revealed the bomb. There had been no previous incidents of persons

A: No, because the rental agency could not have reasonably foreseen the likelihood of someone placing a bomb in the car it was about to rent to the man. A is correct. The standard to be applied in a negligence action is whether the defendant acted with ordinary care. The presence of a bomb in a rental car is sufficiently unlikely that a reasonable rental agency would not routinely inspect for such a device. In the absence of evidence that the agency should have foreseen that there might be a bomb hidden in the car, the man cannot prove a negligence claim.

A young woman who attended a rock concert at a nightclub was injured when the band opened its performance with illegal fireworks that ignited foam insulation in the club's ceiling and walls. The young woman sued the radio station that sponsored the performance. The radio station has moved for summary judgment, claiming that it owed no duty to audience members. The evidence has established the following facts: The station advertised its sponsorship on the radio and in print, distributed free ti

A: No, because there is sufficient evidence of knowledge and control on the part of the station to impose on it a duty of care to audience members. A is correct. A duty of care is owed to only foreseeable plaintiffs. Here, the radio station owed a duty to the attendees of the concert because the station advertised its sponsorship on the radio and in print, distributed free tickets to the concert, and more. The station had sufficient knowledge and control, giving rise to a duty to the concert attendees.

An eight-year-old child went to the grocery store with her mother. The child pushed the grocery cart while her mother put items into it. The child's mother remained near the child at all times. Another customer in the store noticed the child pushing the cart in a manner that caused the customer no concern. A short time later, the cart the child was pushing struck the customer in the knee, inflicting serious injury. If the customer brings an action, based on negligence, against the child, the chi

A: The child exercised care commensurate with her age, intelligence, and experience. A is correct. This choice gives a child-appropriate negligence standard of care. The customer's claim for negligence will be allowed, but the child will only be held to the standard of care expected of "a reasonable child" of the same age, training, maturity, experience, and intelligence.

A mother and her six-year-old child were on a walk when the mother stopped to talk with an elderly neighbor. Because the child resented having his mother's attention diverted by the neighbor, the child angrily threw himself against the neighbor and knocked her to the ground. The neighbor suffered a broken wrist as a result of the fall. In an action for battery by the neighbor against the child, what is the strongest argument for liability? A: The child intended to throw himself against the nei

A: The child intended to throw himself against the neighbor. A is correct. Even though he is a child, he will be liable for the intentional tort of battery if the neighbor can show that the child had the requisite intent, i.e., that he intended to make the offensive or harmful bodily contact.

A company set up a website for the advertisement of goods and services offered by individuals, as well as other public notices. One of the notices on the site announced that the furnishings in a home at a specified address were free for the taking. Within a few hours of the posting, all the furnishings had been taken. The notice had been placed by the homeowner's cousin without the homeowner's knowledge. The cousin bore a grudge against the homeowner and had placed the notice while the homeo

A: The company had no duty to the homeowner. A is correct. The elements of a claim for negligence are duty, breach of duty, causation, and damages. The company's best argument against liability is that it has no duty to protect people from being injured by third parties who intentionally misuse its site, which merely provides a platform for transactions between individuals. Without any duty owed, there can be no finding of negligence.

In a civil action, a plaintiff sued a decedent's estate to recover damages for injuries she suffered in a collision between her car and one driven by the decedent. At trial, the plaintiff introduced undisputed evidence that the decedent's car had swerved across the centerline of the highway into oncoming traffic, where it had collided with the plaintiff's car. The decedent's estate introduced undisputed evidence that, before he swerved across the centerline, the decedent had suffered a f

A: The decedent's estate, because its rebuttal evidence is undisputed. A is correct. The plaintiff's evidence that the decedent violated the statute and crossed over the centerline establishes a prima facie case of negligence. However, the prima facie case of negligence may be rebutted by showing that compliance with the statute was beyond the defendant's control. Here, the decedent's estate successfully rebutted the plaintiff's evidence by providing an undisputed explanation of how the accident happened that is inconsistent with a finding of negligence (the decedent's unforeseeable heart attack made her unable to comply with the statute or, indeed, with any standard of care). B is incorrect. This answer correctly stat

A car driven by the defendant entered land owned by and in the possession of the plaintiff, without the plaintiff's permission. Which, if any, of the following allegations, without additional facts, would provide a sufficient basis for a claim by the plaintiff against the defendant? A: The defendant intentionally drove his car onto the plaintiff's land. B: The defendant's car damaged the plaintiff's land. C: The defendant negligently drove his car onto the plaintiff's land. D: The defendant'

A: The defendant intentionally drove his car onto the plaintiff's land. This is the only choice that, if true, would be sufficient for a claim of trespass to land. The initial facts establish two of the elements, physical invasion and causation, and this answer satisfies the third element - intent. Therefore, if the defendant intentionally drove his car onto the plaintiff's land, this would provide a sufficient basis for a trespass claim.

A driver was traveling along a highway during an unusually heavy rainstorm when the roadway began to flood. To protect his car from water damage, the driver pulled his car up a steep, unmarked driveway abutting the highway that led to a homeowner's residence. The driver left his car parked in the driveway and walked home, intending to return when the floodwater had subsided. Shortly after the driver started to walk home, the homeowner carefully rolled the car back down his driveway and parked

A: Yes, because the driver was privileged to park his car on the homeowner's property. A is correct. The driver's intentional intrusion onto the homeowner's property was indeed a trespass, but the trespass was privileged by the necessity created by the storm. A landowner has no right to forcibly expel a trespasser or a trespasser's property when the trespasser was driven by necessity to trespass on his land, and the landowner is liable for any damage to property of the trespasser that results from an expulsion.

As a seller, an encyclopedia salesman, approached the grounds on which a homeowner's house was situated, he saw a sign that said, "No salesmen. Trespassers will be prosecuted. Proceed at your own risk." Although the seller had not been invited to enter, he ignored the sign and drove up the driveway toward the house. As he rounded a curve, a powerful explosive charge buried in the driveway exploded, and the seller was injured. Can the seller recover damages from the homeowner for his injuries?

A: Yes, because the homeowner was responsible for the explosive charge under the driveway. A is correct. The homeowner is liable for battery because he: (i) committed the required act by placing the explosives under his driveway; and (ii) this caused the harm to the salesman. Causation is still present even though the contact was indirect.

The defendant operates a collection agency. He was trying to collect a valid $400 bill for medical services rendered to the plaintiff by a doctor that was past due. The defendant went to the plaintiff's house and when the plaintiff's mother answered the door, the defendant told her that he was there to collect a bill owed by the plaintiff. The mother told the defendant that because of the plaintiff's illness, the plaintiff had been unemployed for six months, that she was still ill and unable to

A: Yes, because the plaintiff suffered severe emotional distress as a result of the defendant's conduct. A is correct. The defendant, a bill collector, came to the house of the plaintiff, a severely ill person, and loudly threatened her multiple times with criminal fraud charges over the payment of a hospital bill. This would likely constitute extreme and outrageous behavior

The police in a large city notified local gas station attendants that a woman recently had committed armed robberies at five city gas stations. The police said that the woman was approximately 75 years old, had white hair, and drove a vintage, cream-colored Ford Thunderbird. Attendants were advised to call the police if they saw her, but to not attempt to apprehend her. Armed robbery is a felony under state law. A traveler was passing through the city on a cross-country journey. The traveler was

A: not prevail, because the owner reasonably believed that the traveler was the wanted woman. A is correct. Although the owner's actions in keeping the traveler at the gas station satisfy the elements of false imprisonment, the owner will be protected by privilege given that he reasonably believed the traveler had committed a felony, which had actually occurred, and the scope of the confinement was not unreasonable.

A supermarket is in a section of town where there are sometimes street fights and where pedestrians are occasionally the victims of pickpockets and muggers. In recognition of the unusual number of robberies in the area, the supermarket posted signs in the store that read: "Warning: There are pickpockets and muggers at work in this part of the city. The supermarket is not responsible for the acts of criminals." Other than posting the signs, the supermarket took no other precautions to prevent cri

A: plaintiff, because the supermarket failed to take reasonable steps to protect customers against criminal attack in its parking lot. A is correct. The supermarket had a duty to do more than post a warning. The fact that it was aware of ongoing violent crimes in the area meant that it had a duty to take reasonable affirmative steps to keep its customers safe against foreseeable dangers by third persons.

A professional football player signed a written consent for his team's physician to perform a knee operation. After the athlete was under a general anesthetic, the doctor asked a world famous orthopedic surgeon to perform the operation. The surgeon's skills were superior to the doctor's, and the operation was successful. In an action for battery by the athlete against the surgeon, the athlete will A: prevail, because the athlete did not agree to allow the surgeon to perform the operation. B:

A: prevail, because the athlete did not agree to allow the surgeon to perform the operation. A is correct. Battery requires harmful or offensive contact to the plaintiff's person, intent, and causation. Contact to the plaintiff's person is considered offensive if it is not expressly or impliedly consented to by the plaintiff. Unlike a medical malpractice claim, a prima facie case for battery does not require proof of damages to prevail. In this case, the athlete signed a consent form specifically allowing the team doctor to perform the surgery, not the surgeon. Regardless of any superior skill level, the surgeon touched the athlete without his consent, which constitutes offensive contact, and thus meets the requirements for a battery.

A plaintiff and a defendant were in the habit of playing practical jokes on each other on their respective birthdays. On the plaintiff's birthday, the defendant sent the plaintiff a cake containing an ingredient that he knew had, in the past, made the plaintiff very ill. After the plaintiff had eaten a piece of the cake, he suffered severe stomach pains and had to be taken to the hospital by ambulance. On the way to the hospital, the paramedic, who was driving the ambulance, suffered a heart att

A: prevail, because the defendant knew that the cake would be harmful or offensive to the plaintiff. A is correct. The defendant intentionally fed the plaintiff a substance that he knew would make the plaintiff very ill. Despite the red-herring stating that the two "were in a habit of playing practical jokes," this is a battery. A battery is caused by an intentional harmful or offensive contact to the plaintiff's person or an extension thereof, without consent or privilege. The actual touching need not be done personally by the defendant as long as the defendant set into motion an action with purpose or knowledge to a substantial certainty that the offensive or harmful touching would result.

A gardener's backyard, which is landscaped with expensive flowers and shrubs, is adjacent to a golf course. While a golfer was playing golf on the course, a thunderstorm suddenly came up. As the golfer was returning to the clubhouse in his golf cart, lightning struck a tree on the course, and the tree began to fall in the golfer's direction. In order to avoid being hit by the tree, the golfer deliberately steered his cart onto the gardener's property, causing substantial damage to the gardener's

A: prevail, because, although occasioned by necessity, the golfer's entry onto the gardener's property was for the golfer's benefit. A is correct. The golfer has the "incomplete" privilege of private necessity, which allows trespass (without being branded the legal status of trespasser) onto the property of another to avoid a serious personal threat to life or property, but keeps liability for any actual damage caused by the intrusion. The golfer's need to escape a falling tree in a thunderstorm qualifies as an emergency sufficient to invoke a necessity privilege. The privilege of private necessity means that the golfer is only liable for actual damages.

A plaintiff was walking peacefully along a public street when he encountered the defendant, whom he had never seen before. Without provocation or warning, the defendant picked up a rock and struck the plaintiff with it. It was later established that the defendant was mentally ill and suffered recurrent hallucinations. If the plaintiff asserts a claim against the defendant based on battery, which of the following, if supported by evidence, will be the defendant's best defense? A: The defendant

C: The defendant did not know that he was striking a person. C is correct. If the defendant did not know he was striking a person, such a claim would defeat the intent necessary to be liable for a battery, which requires harmful or offensive contact with the plaintiff.

A bank vice president took substantial kickbacks to approve certain loans that later proved worthless. Upon learning of the kickbacks, the bank's president fired the vice president, telling him, "If you are not out of this bank in 10 minutes, I will have the guards physically throw you out." The vice president left at once. If the vice president asserts a claim against the president based on assault, will the vice president prevail? A: No, because the guards never touched the vice president.

B: No, because the president gave the vice president 10 minutes to leave. B is correct. In a claim for assault, the vice president must show that the president had the apparent present physical ability to immediately complete his threatened battery in order for the vice president to have had an apprehension of an imminent harmful or offensive contact. Words alone are not sufficient. Therefore, the president did not commit an assault.

A college student was asleep in his bed in a college dormitory when his roommate, in a drunken fury, entered their room intending to attack the student with an ice pick while he slept. Fortunately, the phone rang and awakened the student. The roommate retreated quickly and threw the ice pick under his own bed in the same room. The next day, the student heard from friends about the roommate's murderous plans and later found the ice pick under the roommate's bed. Even though the college expell

B: No, because the student was not awake when the roommate entered the room and was unaware until later that the roommate was intending to attack him. B is correct. To establish a claim for assault, a plaintiff must demonstrate that he reasonably apprehended that a harmful or offensive touch was imminent. In this case, because he was asleep, the student did not have the apprehension necessary for an assault claim.

A man rented a beach house for a weeklong vacation. On the day he arrived, just after sunset, he took his bag upstairs to a second-floor bedroom and unpacked. As he was about to head back downstairs, he realized that the stairwell had become too dark to navigate without a light. The man spent about 30 seconds feeling the walls at the top of the stairwell but could not find a light switch. In fact, the switch was located in an awkward position not reachable without descending to the second step.

B: "If you conclude that the owner failed to provide reasonably safe premises, but that the man knowingly and voluntarily chose to encounter the risk of falling on the darkened stairs, then you must find for the owner." B is correct. The traditional assumption of risk defense is a complete defense. Therefore, if the jury finds that the man knowingly and voluntarily chose to encounter the risk of falling down the darkened stairs, the owner will not be liable.

A mother took her five-year-old child to a hospital emergency room for treatment. A doctor on the hospital staff molested the child while treating her. At the time, no one was in the treatment room except the doctor and the child; the mother had left the room to get a cup of coffee. Two weeks later, when the child told her mother what had occurred in the treatment room, the mother suffered severe emotional distress that caused her to become physically ill. In an action against the doctor by the

B: No, because the mother was neither the direct victim of the doctor's conduct nor a contemporaneous witness. B is correct. The mother will not succeed in suing the doctor for her emotional distress caused by his intentional harm to her child. Without being a direct victim of his intentional conduct that caused harm, and without any showing that the mother witnessed the harm to her child or even present at the scene, she will have no cause of action.

A driver negligently ran into a pedestrian who was walking along a road. The pedestrian sustained an injury to his knee, causing it to buckle from time to time. Several months later, the pedestrian sustained an injury to his shoulder when his knee buckled, causing him to fall down a flight of stairs. The pedestrian then brought an action against the driver for the injuries to his knee and shoulder. In his action against the driver, for which of his injuries may the pedestrian recover damages?

B: For the injuries to his knee and shoulder, if the jury finds that the pedestrian's fall down a flight of stairs was a normal consequence of his original injury. B is correct. The jury's finding that the fall was a normal consequence of the original injury will allow for the analysis to proceed to a determination of the scope of recoverable damages. Negligent defendants are liable for the full extent damages, regardless of foreseeability.

A man owned a much-loved cat, worth about $25, that frequently trespassed on a neighbor's property. The neighbor repeatedly asked the man to keep the cat on his own property, but the trespasses did not diminish. Aware of the man's long-standing attachment to the cat, the neighbor killed the cat with a shotgun in full view of the man. As a consequence, the man suffered great emotional distress. In an action by the man against the neighbor, which of the following claims would be likely to result i

B: Intentional infliction of emotional distress. B is correct. The tort of intentional infliction of emotional distress allows recovery for personal injury despite the absence of physical injury or touching of the plaintiff. The prima facie elements of a claim for the intentional infliction of emotional distress are: (i) an act by the defendant constituting extreme and outrageous conduct; (ii) intent or recklessness by the defendant; (iii) causation; and (iv) damages amounting to severe emotional distress. On these facts, the neighbor was aware that his conduct would cause severe emotional distress, and he could be held liable for the man's emotional suffering, as well as for the value of the cat.

A patient in a hospital was placed in a wheelchair with his broken leg extended straight out in front of him. As a nurse employed by the hospital was pushing the wheelchair through a set of automatic doors at a normal pace, the doors closed on the patient's foot, injuring it. The nurse attempted to pull the wheelchair back through the doors. This action caused the doors to close more tightly on the patient's foot, injuring it further. The patient sued the hospital, alleging improper maintena

B: No, because a jury could find the hospital liable for negligence based on res ipsa loquitur. B is correct. There is sufficient evidence here for a res ipsa loquitur showing, meaning that a jury could infer that the hospital was probably negligent. As such, summary judgment would be improper based on the existence of a dispute as to a material fact.

Unaware that a lawyer was in the county courthouse library late on a Friday afternoon, when it was unusual for anyone to be using the library, a clerk locked the library door and left. The lawyer found herself locked in when she tried to leave the library at 7 p.m. It was midnight before the lawyer's family could find out where she was and get her out. The lawyer was very annoyed by her detention but was not otherwise harmed by it. Does the lawyer have a viable claim for false imprisonment again

B: No, because the clerk did not intend to confine the lawyer. B is correct. Intent to confine the claimant (or to commit some other intentional tort) is essential to establishing liability for false imprisonment. There is no evidence that the clerk had such an intent.

A host pointed an unloaded revolver at her guest, threatening to shoot him. The guest knew that the revolver was not loaded, and that the ammunition for the revolver was stored in a locked basement closet, two stories below where the two were then standing. In an action brought by the guest against the host for assault, will the guest be likely to prevail? A: No, because the host did not intend to shoot her guest. B: No, because the host did not put her guest in apprehension of an imminent co

B: No, because the host did not put her guest in apprehension of an imminent contact. B is correct. The tort of assault requires that the plaintiff have an apprehension of an imminent bodily contact. That result did not occur here, because the guest knew that the revolver was not loaded and that the ammunition was in a locked basement closet.

A plaintiff suffered from a serious, though not immediately life-threatening impairment of his circulatory system. The plaintiff's cardiologist recommended a cardiac bypass operation and referred the plaintiff to a surgeon. The surgeon did not inform the plaintiff of the 2% risk of death associated with this operation. The surgeon defended his decision not to mention the risk statistics to the plaintiff because the plaintiff "was a worrier and it would significantly lessen his chance of survival

B: No, because the operation was successful and the plaintiff suffered no harm. B is correct. Negligence (including medical malpractice) requires proof of duty, breach, causation, and damages. It is not a dignitary tort. If the plaintiff did not suffer an actual injury, he cannot prevail in an action for negligence against his doctor.

A driver, returning home from a long work shift at a factory, fell asleep at the wheel and lost control of his car. As a result, his car collided with a police car driven by an officer who was returning to the station after having responded to an emergency. The officer was injured in the accident and later sued the driver in negligence for her injuries. The driver has moved for summary judgment, arguing that the common law firefighters' rule bars the suit. Should the court grant the motion?

B: No, because the police officer's injuries were not related to any special dangers of her job. B is correct. The driver could be held liable for his negligence because being struck by a car in normal traffic is not one of the special risks inherent to dangerous police work. The firefighters' rule bars only claims for injuries that result from risks that are unique or special to the plaintiff's inherently dangerous work.

A hiker sustained a head injury when he was struck by a limb that fell from a tree. At the time of his injury, the hiker was walking through a forest on private property without the property owner's knowledge or permission. It was determined that the limb fell because the tree was infested with termites. In an action by the hiker against the property owner to recover for his head injury, will the hiker be likely to prevail? A: No, because the property owner could not foresee that anyone woul

B: No, because the property owner breached no duty to the hiker, who was a trespasser. B is correct. The hiker was a trespasser because he entered without the permission of the owner. A possessor of land is not required to exercise reasonable care to make his land safe for trespassers

During a comprehensive evaluation of an adult patient's psychiatric condition, a psychiatrist failed to diagnose the patient's suicidal state. One day after the misdiagnosis, the patient committed suicide. The patient's father, immediately after having been told of his son's suicide, suffered severe emotional distress, which resulted in a stroke. The patient's father was not present at the patient's appointment with the psychiatrist, nor did he witness the suicide. The father has bro

B: No, because the psychiatrist's professional duty did not extend to the harms suffered by the patient's father. B is correct. Courts do not impose a duty on medical professionals to warn third parties when a patient is a danger to himself and not others and will deny liability to family members who suffered emotionally in the event that the patient commits suicide.

A vintner is the owner of a large vineyard and offers balloon rides to visitors who wish to tour the grounds from the air. During one of the rides, the vintner was forced to make a crash landing on his own property due to high winds. Without the vintner's knowledge or consent, a trespasser had entered the vineyard to camp for a couple of days. The trespasser was injured when he was hit by the basket of the descending balloon. If the trespasser sues the vintner to recover damages for his injuries

B: No, because the vintner was unaware of the trespasser's presence until after the injury had occurred. B is correct. The vintner has no duty to an undiscovered trespasser. If the vintner knew or reasonably should have known of the trespasser's presence, under majority law he has a duty of reasonable care to avoid injury to the trespasser. (Under traditional common law a discovered trespasser was only owed the duty to avoid gross negligence or willful and wanton misconduct.) This choice appropriately addresses the issue of awareness and the corresponding duty.

A customer fell and injured himself when he slipped on a banana peel while shopping at a grocery store. The banana peel was fresh and unblemished except for a mark made by the heel of the customer's shoe. In an action brought by the customer against the store, these are the only facts in evidence. Should the trial judge permit the case to go to the jury? A: No, because the customer had an obligation to watch where he stepped. B: No, because there is not a reasonable basis for inferring that

B: No, because there is not a reasonable basis for inferring that the store knew or should have known of the banana peel. B is correct. Unlike slip-and-fall cases in which res ipsa loquitur is appropriate, the condition of the banana peel does not indicate that it has been on the ground for any significant period of time. Therefore, there is not enough evidence to support a jury verdict that the store staff was negligent in failing to remove it before the customer's fall.

A complaint filed on behalf of a woman against a nursing home and an ambulance service included the following allegations: The woman, who was 86 years old and unable to speak after suffering a stroke, was picked up from her daughter's house by the ambulance service and taken to the nursing home to stay while her daughter was out of town. When the woman's daughter returned a few days later, the ambulance service picked up the woman from the nursing home and returned her to the daughter's ho

B: One of the two defendants probably caused the injury, and the circumstances of the injury are primarily within the knowledge and control of the defendants rather than the woman or her representative. B is correct. The physician's report stated that the woman's leg injury would not have occurred in the absence of negligence. However, the woman had suffered a stroke and was unable to speak. Therefore, the defendants are the ones with primary knowledge and control of what caused the injury. The woman cannot offer any more allegations in her complaint.

In a plaintiff's action for battery, the evidence established the following: the plaintiff was bad-tempered and, as the defendant knew, carried a gun and used it often; the plaintiff struck the defendant first; during the altercation, the plaintiff repeatedly tried to get to his gun; and the blows inflicted upon the plaintiff by the defendant resulted in the plaintiff being hospitalized. Which finding of fact would be most likely to result in a verdict for the defendant? A: The defendant use

B: The defendant used no more force than he reasonably believed was necessary to protect himself against death or serious bodily harm. B is correct. The privilege of self-defense permits the use of force actually and reasonably believed to be necessary given the threat posed by the plaintiff.

A pedestrian was crossing a street at a crosswalk. A bystander, who was on the sidewalk nearby, thought he saw a speeding automobile heading in the pedestrian's direction. However, the automobile was obviously coming to a stop at the traffic light. Nevertheless, the bystander ran into the street and pushed the pedestrian onto the sidewalk. The pedestrian fell to the ground and broke her leg. In an action for battery brought by the pedestrian against the bystander, will the pedestrian prevail?

B: Yes, because the pedestrian was not actually in danger and the bystander should have realized it. B is correct. The bystander is liable for pushing the pedestrian because his mistake regarding whether the pedestrian was in danger was unreasonable. The fact that the car was obviously coming to a stop means that it was unreasonable to believe the pedestrian was in danger. The bystander thus has no defense and will be liable for battery.

The day after a seller completed the sale of his house and moved out, one of the slates flew off the roof during a windstorm. The slate struck a pedestrian who was on the public sidewalk. The pedestrian was seriously injured. The roof is old and has lost several slates in ordinary windstorms on other occasions when the seller was present. The pedestrian was also aware that past windstorms had blown slates off the roof. If the pedestrian sues the seller to recover damages for his injuries, will t

B: Yes, because the seller should have been aware of the condition of the roof and should have realized that it was dangerous to persons outside the premises. B is correct. The seller had a duty of reasonable care for the safety of those outside the land to prevent harm resulting from conditions on the land. The facts indicate that ordinary windstorms had previously caused tile loss. Despite the fact that the seller just sold the home, he was or should have been aware of the dangerous condition of the roof and the foreseeability of harm to persons outside the premises from windswept falling tiles. Instead, he left the dangerous conditions without taking precautionary measures to protect against dislodged tiles.

A neighbor, who lived next door to a homeowner, went into the homeowner's garage without permission and borrowed the homeowner's chainsaw. The neighbor used the saw to clear broken branches from the trees on the neighbor's own property. After he had finished, the neighbor noticed several broken branches on the homeowner's trees that were in danger of falling on the homeowner's roof. While the neighbor was cutting the homeowner's branches, the saw broke. In a suit for conversion by the homeowner

B: Yes, for the value of the saw before the neighbor borrowed it. B is correct. The call of the question gives a claim for conversion. Conversion occurs when the defendant's trespass on the plaintiff's property interest is substantial and amounts to an act of ownership/dominion.

A defendant and a group of his friends are fanatical basketball fans who regularly meet at each others' houses to watch basketball games on television. Some of the group are fans of the home team, and others are fans of the rival team. When the group has watched televised games between these two teams, fights sometimes have broken out among the group. Despite this fact, the defendant invited the group to his house to watch a championship game between the home team and the rival team. During the

B: a police officer is not entitled to a recovery based upon the negligent conduct that created the need for the officer's professional intervention. B is correct. The "firefighter's rule" bars from recovery a firefighter or police officer who was injured by a peril that he was employed to confront. If the peril was created through negligence or caused by a strict liability activity, the officer generally still has no claim against the landowner (the defendant). Here, the police officer was called to the house with the expectation that he would be confronting the fight, rendering this the most effective argument in favor of the motion.

While a driver was taking a leisurely spring drive, he momentarily took his eyes off the road to look at some colorful trees in bloom. As a result, his car swerved a few feet off the roadway, directly toward a pedestrian, who was standing on the shoulder of the road waiting for a chance to cross. When the pedestrian saw the car bearing down on him, he jumped backwards, fell, and injured his knee. The pedestrian sued the driver for damages, and the driver moved for summary judgment. The foregoing

B: denied, because a jury could find that the driver negligently caused the pedestrian to suffer a legally compensable injury. B is correct. A motion for summary judgment will be granted where there is no genuine issue of material fact and the moving party is entitled to judgment as a matter of law. A question of fact exists as to whether the driver's negligence caused the pedestrian's reaction, or whether the pedestrian's injury was the result of his own act. A motion for summary judgment by the driver can only be rendered for the driver or denied.

A defendant negligently caused a fire in his house, and the house burned to the ground. As a result, the sun streamed into the plaintiff's yard next door, which previously had been shaded by the defendant's house. The sunshine destroyed some delicate and valuable trees in the plaintiff's yard that could grow only in the shade. The plaintiff has brought a negligence action against the defendant for the loss of the plaintiff's trees. The defendant has moved to dismiss the complaint. The best argum

B: the defendant's duty to avoid the risks created by a fire did not encompass the risk that sunshine would damage the plaintiff's trees. B is correct. It states the appropriate standard at issue. In an action for negligence, the defendant has a duty only to act in a reasonably prudent manner with regard to foreseeable harm. When the fire was negligently started in his own home, the defendant's duty to avoid foreseeable risks posed by a fire arguably would not have included sun damage to his neighbor's trees. In a motion to dismiss, if the plaintiff cannot show foreseeable risk, then there is no duty and no claim for negligence.

A driver negligently drove his car into a pedestrian, breaking her leg. The pedestrian's leg was put in a cast, and she used crutches to get around. While shopping at her local supermarket, the pedestrian non-negligently placed one of her crutches on a banana peel that had been negligently left on the floor by the manager of the supermarket's produce department. The pedestrian's crutch slipped on the peel, and she fell to the floor, breaking her arm. Had the pedestrian stepped on the banana peel

B: the driver, for both of her injuries. B is correct. The pedestrian's broken arm is proximately connected to the driver's negligent driving. Courts generally hold that subsequent ordinary negligence actions, which may add to the plaintiff's injuries, are a foreseeable consequence of the original action and will not break the chain of causation. The facts clearly state that the pedestrian would not have lost her balance had she not been on crutches as the result of being hit by the driver. Because it is foreseeable that walking on crutches would make the pedestrian less stable on her feet and more vulnerable to subsequent injury from falling, the driver is the proximate (legal) cause of both injuries (although responsibility for the broken arm alone may be apportioned between the driver and the supermarket as concurrent tortfeasors).

While an equestrian was riding her horse on what she thought was a public path, the owner of a house next to the path approached her, shaking a stick and shouting, "Get off my property." Unknown to the equestrian, the path on which she was riding crossed the private property of the shouting owner. When the equestrian explained that she thought the path was a public trail, the man cursed her, approached the equestrian's horse, and struck the horse with the stick. As a result of the blow, the hors

B: the equestrian, for battery and assault. B is correct. The equestrian suffered two specific tort injuries. The first was assault. For assault, the defendant must have the apparent present physical ability to complete his threatened battery for the tort of assault to be complete. Words alone are not sufficient. This first tort occurred when the owner approached her, yelling and shaking a stick at her. The second was a battery. A battery is caused by an intentional harmful or offensive touch to the plaintiff's person or an extension thereof, without consent or privilege. When the owner struck the horse the equestrian was seated on, he committed a battery by striking an extension of the equestrian, causing an offensive touch. Choice B appropriately lists both torts.

The manager of a department store noticed that a customer was carrying a scarf with her as she examined various items in the blouse department. The manager recognized the scarf as an expensive one carried by the store. The customer was trying to find a blouse that matched a color in the scarf, and, after a while, found one. The manager then saw the customer put the scarf into her purse, pay for the blouse, and head for the door. The manager, who was eight inches taller than the customer, blocked

B: the manager had a reasonable belief that the customer was shoplifting and detained her only briefly for a reasonable investigation of the facts. B is correct. The manager had a "shopkeeper's (or merchant's) privilege" to detain the customer. For the privilege to be valid, the manager must have had reasonable grounds to believe that the customer was stealing or attempting to steal store property, the detention must be for a reasonable period of time, and it must be conducted in a reasonable manner. If the manager had held the customer after the issue of the stolen scarf had been settled, or after the goods had been recovered in an attempt to obtain a signed confession, the customer's claim for false imprisonment would prevail. B is the best defense because it addresses the elements of the privilege as they pertain to the facts of the situation.

A man was admitted to a hospital after complaining of persistent severe headaches. While he was there, hospital staff failed to diagnose his condition, and he was discharged. Two days later, the man died of a massive brain hemorrhage due to a congenital defect in an artery. The man's wife has brought a wrongful death action against the hospital. The wife offers expert testimony that the man would have had a "reasonable chance" (not greater than 50%) of surviving the hemorrhage if he had been giv

C: A jurisdiction that allows recovery for the loss of the chance of survival. C is correct. In establishing a claim of negligence against a defendant, a plaintiff must prove that the defendant's action was the factual cause of the harm suffered. This becomes difficult to prove when the victim likely would have suffered the harm (death in this example) even if the defendant had acted reasonably. In jurisdictions that allow recovery for loss of chance of survival, courts allow plaintiffs to recover a portion of his or her damages for the reduction in survival chance.

A car owner washed her car while it was parked on a public street, in violation of a local ordinance that prohibits the washing of vehicles on public streets during specified hours. The ordinance was enacted only to expedite the flow of automobile traffic. Due to sudden and unexpected cold weather, the car owner's waste water formed a puddle that froze in a crosswalk. A pedestrian slipped on the frozen puddle and broke her leg. The pedestrian sued the car owner to recover for her injury. At tr

C: Grant the car owner's motion, because the pedestrian has failed to offer adequate evidence that the car owner was negligent. C is correct. The pedestrian offered no evidence supporting the claim of negligence except the ordinance violation. The ordinance was not adopted to reduce the risk of accumulating ice on public walkways. Accordingly, the car owner's motion should be granted.

A company designed and built a processing plant for the manufacture of an explosive chemical. An engineer was retained by the company to design a filter system for the processing plant. She prepared an application for a permit to build the plant's filter system and submitted it to the state's Department of Environmental Protection (DEP). As required by DEP regulations, the engineer submitted a blueprint to the DEP with the application for permit. The blueprint showed the entire facility and was

C: No, because the engineer owed no duty to the plaintiff to prevent the particular risk of harm. C is correct. While this looks like a products liability question, it is a professional malpractice issue. The company designed and built the processing plant. The engineer was retained solely for the purpose of designing a filter system for the plant. She had a duty to exercise skill in the design of the filter system, commensurate with her professional training and standards. The engineer's use of the company's blueprint for the proper permit did not impute liability onto the engineer for the entire facility, as a permit is nothing more than a license (permission) to proceed with construction, not a guarantee against defect.

In the course of a bank holdup, a robber fired a gun at a guard. The guard drew his revolver and returned fire. One of the bullets fired by the guard ricocheted, striking the plaintiff, who was simply a customer at the bank. If the plaintiff asserts a claim against the guard based upon battery, will the plaintiff prevail? A: Yes, because the plaintiff was not the robber's accomplice. B: Yes, under the doctrine of transferred intent. C: No, because the guard fired reasonably in his own defens

C: No, because the guard fired reasonably in his own defense. C is correct. The guard will prevail against the plaintiff's battery claim because the guard reasonably acted in selfdefense by returning fire when the robber shot at him. Even though the guard injured the plaintiff when he fired, and the plaintiff was an innocent bystander, the guard's self-defense privilege will protect him from liability because he injured the plaintiff by accident.

hiker, although acting with reasonable care, fell while attempting to climb a mountain and lay unconscious and critically injured on a ledge that was difficult to reach. The plaintiff, an experienced mountain climber, was himself seriously injured while trying to rescue the hiker. The plaintiff's rescue attempt failed, and the hiker died of his injuries before he could be reached. The plaintiff brought an action against the hiker's estate for compensation for his injuries. In this jurisdiction,

C: No, because the hiker's peril did not arise from his own failure to exercise reasonable care. C is correct. One who acts negligently and endangers himself is also liable for the resulting injuries of anyone who undertakes to rescue him. Here, however, the hiker was not negligent in creating his peril, and the plaintiff will not prevail.

A schizophrenic patient who was institutionalized in a psychiatric facility pushed a nurse down a stairwell at the facility. The nurse, a paid employee of the facility who was trained to care for schizophrenic patients, was injured. The patient is an indigent whose care is paid for by the government. The jurisdiction generally follows the rule that a person with a mental deficiency is held to the standard of a reasonable person. In a negligence action brought by the nurse against the patient, th

D: The patient is an indigent whose care is paid for by the government. D is correct. Whether the patient has the resources to satisfy an adverse judgment is irrelevant to the judgment itself and should not be a subject of argument on the issue of liability in the case, although the patient's financial situation might affect a lawyer's decision to take on the case.

A child was bitten by a dog while playing in a fenced-in common area of an apartment complex owned by a landlord. The child was the guest of a tenant living in the complex, and the dog was owned by another tenant. The owner of the dog knew that the dog had a propensity to bite, but the landlord did not have any notice of the dog's vicious propensities. In an action by the child against the landlord, will the child be likely to prevail? A: Yes, because in these circumstances a landlord is str

C: No, because the landlord did not have notice of the dog's vicious propensities. C is correct. Any duty that the landlord may have is at most a duty to act reasonably. Because the landlord had no reason to know that the dog posed a risk to those on his property, his failure to take precautions against that risk was not negligent.

A real estate developer was trying to purchase land on which he intended to build a large commercial development. An elderly widow had rejected all of the developer's offers to buy her ancestral home, where she had lived all her life and which was located in the middle of the developer's planned development. Finally, the developer offered her $250,000. He told her that if she rejected it, state law authorized him to have her property condemned. He subsequently parked a bulldozer in front of her

C: No, because the widow did not suffer emotional distress that was severe. C is correct. The widow being badly frightened and outraged is insufficient to establish that she experienced severe emotional distress. She sought no medical attention and offered no additional evidence that the level of distress she experienced necessitated medical attention.

The plaintiff, a jockey, was seriously injured in a race when another jockey, the defendant, cut too sharply in front of her without adequate clearance. The two horses collided, causing the plaintiff to fall to the ground, sustaining injury. The State Racetrack Commission ruled that, by cutting in too sharply, the defendant committed a foul in violation of racetrack rules requiring adequate clearance for crossing lanes. The plaintiff has brought an action against the defendant for damages in whi

C: No, unless the defendant intended to cause impermissible contact between the two horses or apprehension of such contact by the plaintiff. C is correct. If the plaintiff does not establish the defendant's intent in one of these two ways, she will not prevail. She must show that the defendant either intended to make offensive contact with the plaintiff's person or intended to commit assault in that the defendant intended to place her in apprehension of immediate harmful or offensive contact.

A bright 12-year-old child attended a day-care center after school. The center was located near a man-made duck pond on the property of a corporation. During the winter, the pond was used for ice-skating when conditions were suitable. At a time when the pond was obviously only partially frozen, the child sneaked away from the center's property and walked out onto the ice over the pond. The ice gave way, and the child fell into the cold water. He suffered shock and would have drowned had he not

C: The day-care center, because it was not negligent. C is correct. Here, the call of the question states the suit is between the day-care center and the child. However, the facts state that the pond in question was the property of the corporation. Thus, the attractive nuisance doctrine would not apply under these facts against the day-care center. Thus, there is no evidence of lack of reasonable care by the day-care center and they would likely prevail.

A bright 12-year-old child attended a day-care center after school. The day-care center was located near a man-made duck pond on the property of a corporation. During the winter, the pond was used for ice-skating when conditions were suitable. At a time when the pond was obviously only partially frozen, the child sneaked away from the center's property and walked out onto the ice over the pond. The ice gave way, and the child fell into the cold water. He suffered shock and would have drowned h

C: The corporation, because the danger of thin ice may reasonably be expected to be understood by a 12- year-old child. D: The corporation, because the day-care center had a duty to keep the child off the ice. C is correct. The child-trespasser was a bright 12-year-old, which means he should have appreciated the magnitude of the risk in light of the sign explicitly warning him from not going onto the ice.

Under the Federal Tort Claims Act, with certain exceptions not relevant here, the federal government is liable only for negligence. A federally owned and operated nuclear reactor emitted substantial quantities of radioactive matter that settled on a nearby dairy farm, killing the dairy herd and contaminating the soil. At the trial of an action brought against the federal government by the farm's owner, the trier of fact found the following: (1) the nuclear plant had a sound design, but a valve

C: The government, on the ground that a case under the Federal Tort Claims Act has not been proved. C is correct. There has been no finding of negligence on the part of the government. The trier of fact found that the government had selected a reliable manufacturer for the component part and could not have anticipated or prevented the malfunction. The court should therefore enter judgment for the defendant, on the ground that a case under the Federal Tort Claims Act has not been proved.

A recently installed elevator suddenly started free-falling down the elevator shaft while carrying passengers. Frightened, a passenger pried the inside doors open and impulsively stuck his arm through them to try to stop the fall. As a result, his arm was broken. The elevator eventually stopped without causing further injuries. In a negligence action brought by the injured passenger against the company that installed and maintained the elevator, the injured passenger has asked the trial judge to

C: The judge should grant the passenger's request but should also instruct the jurors to consider any carelessness of the passenger in awarding damages if they find the company liable. C is correct. The question of the passenger's negligence is relevant to the jury's assessment of damages but is not a bar to a res ipsa loquitur instruction. Under res ipsa, where the facts strongly indicate that the plaintiff's injuries resulted from the defendant's negligence, the trier of fact may be permitted to infer that the defendant was probably negligent. This approach allows the jury to skip over the typical negligence analysis of duty and breach of duty and allows the defendant's negligence to be presumed.

In an action by a man against a pharmacy, the man offered only the following evidence: The man took a clearly written prescription to a pharmacy. The pharmacy's employee filled the prescription by providing pills with 30 milligrams of the active ingredient instead of 20 milligrams, as was prescribed. Shortly after taking the pills as directed, the man, who had no previous history of heart problems, suffered a heart attack. Overdoses of the active ingredient had previously been associated with

C: Yes, because a jury could reasonably conclude that the man would not have suffered a heart attack had the pharmacy provided the correct dosage. C is correct. There is sufficient circumstantial evidence to support a conclusion that the pharmacy's employee was negligent in filling the prescription and that the consequent overdose caused the heart attack. The pharmacy would be vicariously liable for its employee's negligence under respondeat superior principles.

A customer bought a can of corn at a grocery store. While eating the corn later that evening, the customer was injured by a small piece of glass in the corn. The customer sued the canning company that had processed and canned the corn. At trial, the customer presented evidence that neither the customer nor any third party had done anything after the can of corn was opened that would account for the presence of the glass. Without any other evidence, is the customer likely to prevail? A: No, bec

C: Yes, because a jury may reasonably infer that the canning company acted negligently. C is correct. Under the doctrine of res ipsa loquitur, courts can infer negligence from the very nature of an accident or injury in the absence of direct evidence on how the defendant behaved. The elements of duty, breach, and causation are inferred from the injury because it would not ordinarily occur without negligence. The customer presented evidence that neither the customer nor any third party had done anything with the can after it was opened that would account for the glass. Therefore, the jury can infer that if not for negligence on behalf of the canning company, the injury would not have occurred.

A 14-year-old girl of low intelligence received her parents' permission to drive their car. She had had very little experience driving a car and did not have a driver's license. Although she did the best she could, she lost control of the car and hit a pedestrian. The pedestrian has brought a negligence action against the girl. Is the pedestrian likely to prevail? A: No, because only the girl's parents are subject to liability. B: No, because the girl was acting reasonably for a 14-year-old o

C: Yes, because the girl was engaging in an adult activity. D: Yes, because the girl was not old enough to obtain a driver's license. C is correct. The girl was engaging in a dangerous activity that is characteristically undertaken by adults, so she will be held to the adult standard of care. No adjustment will be made to that standard to reflect her low intelligence and lack of experience. Her low intelligence and her inexperience put others at risk, and she will be held to the standard of a reasonably prudent driver even if she is not capable of reasonable prudence.

A man's car sustained moderate damage in a collision with a car driven by a woman. The accident was caused solely by the woman's negligence. The man's car was still drivable after the accident. Examining the car the next morning, the man could see that a rear fender had to be replaced. He also noticed that gasoline had dripped onto the garage floor. The collision had caused a small leak in the gasoline tank. The man then took the car to a mechanic, who owns and operates a body shop, and arranged

C: Yes, because a reasonable person in the man's position would have warned the mechanic about the gasoline leak. C is correct. The man's conduct is subject to the objective reasonable person standard. Therefore, the man had a duty to inform the mechanic of the leak, because a reasonably prudent person in the man's position would have given a warning.

A security guard, dressed in plain clothes, was working for a discount store when a customer got into a heated argument with a cashier over the store's refund policy. Without identifying himself as a security guard, the security guard suddenly grabbed the customer's arm. The customer attempted to push the security guard away, and the security guard knocked the customer to the floor, causing injuries. The customer sued the discount store for battery on a theory of vicarious liability for the

C: Yes, because contributory negligence is not a defense to battery. C is correct. Contributory negligence is a defense to negligence. Here, the action is for battery, an intentional tort. Therefore, the defense will not be available.

A doctor ordered chest X-rays for a patient who smoked cigarettes. After the consulting radiologist told the doctor that the X-rays looked normal, the doctor told the patient that he was in good health. In fact, the radiologist had missed signs of cancer on the X-rays that a trained radiologist, acting competently, would have detected. After another X-ray of the patient's chest, performed one year later, showed advanced lung cancer, the doctor discovered that the radiologist had misinterpreted

C: Yes, because punitive damages awards are not authorized unless there is proof of willful or wanton misconduct on the defendant's part. C is correct. Punitive damages may be awarded where there is proof of the defendant's willful or wanton misconduct. Although the radiologist was negligent, nothing in the facts provided suggests that the radiologist acted with willful or wanton misconduct. Thus, the radiologist is likely to have the punitive damages award vacated on appeal.

A defendant built in his backyard a garage that encroached two feet across the property line onto property owned by his neighbor. Thereafter, the defendant sold his property to a friend. The neighbor was unaware, prior to the defendant's sale to his friend, of the encroachment of the garage onto her property. When the neighbor subsequently learned of the encroachment, she sued the defendant for damages for trespass. In this action, will the neighbor prevail? A: No, because the defendant was una

C: Yes, because the defendant knew where the garage was located, whether or not he knew where the property line was. C is correct. The defendant's intentional entry onto the land was enough to satisfy intent; he did not need to know he was trespassing onto the neighbor's land to be held liable.

A man and a woman were competing in an illegal drag race. Both of them were driving over the speed limit but were otherwise driving very carefully. However, when a tire on the woman's car suddenly blew out, she lost control of her car and crashed, injuring a pedestrian. The pedestrian later sued the man, because the woman had no insurance or assets. Will the pedestrian be likely to prevail in that action? A: No, because the man did not cause the injury. B: No, because the man was driving ve

C: Yes, because the man and the woman were acting in concert in a dangerous activity. C is correct. When two or more tortfeasors act in concert and injure a plaintiff, then each will be jointly and severally liable for the entire injury. Moreover, anyone who is found to have been engaging in an abnormally dangerous activity will be strictly liable for any injuries which result from the activity. Here, the man and woman were competing in an illegal drag race and were driving over the speed limit. They were acting in concert by drag racing. The man will, therefore, be liable for the pedestrian's injuries.

A patient who had suffered a severe fracture of her leg was treated by an orthopedist, who set the patient's leg and put it in a cast. When the leg continued to bother the patient six months later, she consulted a second orthopedist in the same town. The second orthopedist surgically inserted a pin to facilitate healing. The patient brought a malpractice action against the first orthopedist, claiming that he should have surgically inserted a pin at the time of initial treatment. The only evide

C: Yes, because the patient has failed to introduce evidence that the first orthopedist's care fell below the professional standard of care. C is correct. Professionals are held to a different standard of conduct than that of the ordinary person. Doctors especially have a specialized standard of care; most courts will apply a national standard of care to evaluate their conduct. The patient is responsible for introducing evidence to show that the orthopedist breached his standard of care. There is nothing here to suggest that this was the case.

A pilot was flying his small plane when he experienced engine trouble and was forced to make an emergency landing. He landed the plane safely in a large yard behind a home located in a relatively remote area. Unfortunately, when he disembarked from the plane, he was attacked and injured by two large dogs kept by the homeowner to discourage trespassers. The homeowner, who had seen the plane land, had ordered the dogs to attack. Several months earlier, the homeowner had posted large signs around t

C: Yes, because the pilot can invoke the privilege of necessity. C is correct. A battery occurs when a defendant brings about harmful or offensive contact upon the plaintiff with the intent to do so. The homeowner ordered the dogs to attack and so caused the harmful contact to occur with the requisite intent. Therefore, the pilot has a case for battery. The homeowner may then seek to assert a defense of property defense against the pilot, which is a defense to intentional torts. However, when an actor has a privilege to enter the land of another because of necessity, that privilege supersedes the privilege a landowner has in defense of property. Because the pilot had the privilege of necessity, which would defeat the homeowner's defense of defense of property, the pilot can bring a viable battery claim.

A defendant left her car parked on the side of a hill. Two minutes later, the car rolled down the hill and struck and injured the plaintiff. In the plaintiff's negligence action against the defendant, the plaintiff introduced into evidence the facts stated above, which are undisputed. The defendant testified that, when she parked her car, she turned the front wheels into the curb and put on her emergency brakes, which were in good working order. She also introduced evidence that, in the weeks be

C: denied, because, given the defendant's evidence, the jury was not required to draw an inference of negligence from the circumstances of the accident. C is correct. The jury's verdict was reasonable in light of the defendant's evidence countering the plaintiff's res ipsa loquitur showing, which would have allowed an inference of negligence. The defendant offered plenty of evidence to defeat an inference of her negligence, and the motion should be denied.

A traveler was a passenger on a commercial aircraft owned and operated by an airline. The aircraft crashed into a mountain, killing everyone on board. The flying weather was good. The traveler's legal representative brought a wrongful death action against the airline. At trial, the legal representative offered no expert or other testimony as to the cause of the crash. On the airline's motion to dismiss at the conclusion of the legal representative's case, the court should A: grant the motion,

C: deny the motion, because the jury may infer that the aircraft crashed due to the airline's negligence. C is correct. The evidence is sufficient to establish a res ipsa loquitur showing, which allows the trier of fact to infer the airline's negligence. For this reason, the airline's motion to dismiss should be denied because the traveler's legal representative has presented a claim upon which relief can be granted.

While approaching an intersection with the red light against him, a motorist suffered a heart attack that rendered him unconscious. The motorist's car struck a child, who was crossing the street with the green light in her favor. Under the state motor vehicle code, it is an offense to drive through a red traffic light. The child sued the motorist to recover for her injuries. At trial it was stipulated that: (1) immediately prior to suffering the heart attack, the motorist had been driving within

C: grant the motorist's motion, because he had no history of heart disease or warning of the heart attack. C is correct. The stipulated facts prove that there was no legally sufficient evidentiary basis for a reasonable jury to find that the motorist breached his duty of care given that he was not driving negligently and had no history of heart disease that created a foreseeable risk of harm to others while driving.

A plaintiff and a man were passengers sitting in adjoining seats on a flight on an airline. There were many empty seats on the aircraft. During the flight, a flight attendant served the man nine drinks. As the man became more and more obviously intoxicated and attempted to engage the plaintiff in a conversation, the plaintiff chose to ignore the man. This angered the man, who suddenly struck the plaintiff in the face, giving her a black eye. If the plaintiff asserts a claim for damages against t

C: not prevail, because the man was not acting as an agent or employee of the airline. C is correct. For the airline to be held responsible for the man's commission of battery against the plaintiff, vicarious liabillity must apply. However, there is no evidence of a special relationship between the man and the airline, such as the man being an agent or employee. As such, vicarious liability will not attach.

A patient had been under the care of a cardiologist for three years prior to submitting to an elective operation that was performed by a surgeon. Two days thereafter, the patient suffered a stroke, resulting in a coma, caused by a blood clot which formed after the operation. When it appeared that she had entered a permanent vegetative state, with no hope of recovery, the artificial life-support system that had been provided was withdrawn, and she died a few hours later. The withdrawal of artific

C: not prevail, because there is no evidence that a cardiologist would have provided advice that would have changed the outcome if one had examined the patient before the operation. C is correct. The critical issue in this question is whether the lack of a cardiology review is the cause in fact and the legal cause of the injury the patient suffered. This answer choice is the only answer that addresses the need to provide evidence of the causal link between the surgeon's breach of duty and the subsequent medical injury to the patient.

An eight-year-old child had a habit of riding his bicycle onto a busy highway. His parents knew about this habit but continued to let the child ride his bicycle. One afternoon, the child rode his bicycle down his driveway onto the busy highway and a driver had to stop her car suddenly to avoid colliding with the bike. Because of the sudden stop, the driver's son, who was sitting on the seat without any restraint, was thrown into the dashboard and injured. Had the driver's son been properly restr

C: prevail, because the child's parents knew that he sometimes rode into the highway, and they took no steps to prevent it. C is correct. Parents have a number of affirmative duties, based on their special relationship to their minor children. This includes the duty to exercise reasonable care in the control of the parent's minor children. Liability is generally limited to actions that were foreseeable by the parent.

A patron ate a spicy dinner at a restaurant on Sunday night. He enjoyed the food and noticed nothing unusual about the dinner. Later that evening, the patron had an upset stomach. He slept well through the night, went to work the next day, and ate three meals. His stomach discomfort persisted, and by Tuesday morning he was too ill to go to work. Eventually, the patron consulted his doctor, who found that the patron was infected with a bacterium that can be contracted from contaminated food. Food

C: the patron has failed to establish that the restaurant's food caused his illness. C is correct. The defense with the best chance of prevailing. Health code violations can only establish duty and breach; they do not establish the restaurant's causal control over the specific instrumentality that caused the actual food poisoning. In addition, res ipsa loquitur does not apply because the patron was unable to show that the restaurant had exclusive control over everything the patron ate within the period leading up to his illness. As a consequence, the restaurant's negligence cannot be inferred by the circumstances.

A customer pledged a stock certificate to a bank as security for a loan. A year later, when the customer fully repaid the loan, the bank refused the customer's demand to return the stock certificate because the officer dealing with the loan had the mistaken belief that there was still a balance due. No one at the bank reviewed the records until two months later, at which time the error was discovered. The bank then offered to return the stock certificate. However, the customer refused to accep

D: $20,000, because that was the value of the shares when the customer was entitled to the return of the certificate. D is correct. Conversion occurs when a defendant intentionally commits an act depriving the plaintiff of possession of her chattel or interferes with the plaintiff's chattel in a manner so serious as to deprive the plaintiff use of the chattel. The damages are the full value at the time of the conversion. The conversion occurred when the bank refused to relinquish the stock certificate in response to the customer's lawful demand, and at that time the shares were worth $20,000.

While visiting at his son's home, a grandfather tripped on a toy left on the floor by his four-year- old grandson. The grandfather fell and was severely injured. The grandfather regularly visited his son's home and was aware that the grandson routinely left toys scattered about the house. The son had never warned the grandfather to look out for toys. The grandfather brought an action against his son to recover for his injuries, and both the grandfather and the son have moved for directed verdict

D: Grant the son's motion, because the son had no duty to warn that the grandson might leave toys on the floor. D is correct. The duty owed by an owner or occupier of land to those on the premises for dangerous conditions on the land depends on the legal status of the plaintiff with regard to the property. Here, the grandfather was invited to the son's home, making him a licensee. Typically social guests are considered licensees. An owner owes a duty to warn or make safe dangerous known conditions that create an unreasonable risk of harm to the licensee and that the licensee is unlikely to discover. In this case, the grandfather regularly visits the son's home and was aware that toys were typically left around the house. The son had no duty to warn the grandfather because the dangerous condition was one already known to the grandfather. A is incorrect. The motion should be granted

The warden of a state prison prohibits the photographing of the face of any prisoner without the prisoner's consent. A news photographer wanted to photograph a notorious mobster incarcerated at the state prison. To circumvent the warden's prohibition, the photographer flew over the prison exercise yard and photographed the mobster. A prisoner, who was imprisoned for a technical violation of a regulatory statute, happened to be standing next to the mobster when the photograph was taken. When the

D: No reasonable person could conclude that the photographer's conduct was extreme and outrageous as to the prisoner. D is correct. A motion for summary judgment will be granted where there is no genuine issue of material fact and the moving party is entitled to judgment as a matter of law. For the prisoner to prevail in his claim for intentional and reckless infliction of emotional distress, he must show that: (i) the photographer's intentional (with purpose or knowledge to a substantial certainty) or reckless disregard for the consequences of publishing the photograph was; (ii) extreme and outrageous conduct; which (iii) caused the prisoner severe emotional distress. This answer choice addresses the element of outrageousness that is not answered in the fact pattern and which is an objective standard that can be determined as a matter of law.

An intoxicated man who was standing on a fifth-floor apartment balcony threatened to jump off the building. A bystander pulled the man back into the building, pushed him into a bedroom, and locked the bedroom door from the outside. When the man became sober, the bystander released him from the bedroom. Does the man have a claim against the bystander? A: Yes, for battery, because the bystander pushed the man into the bedroom. B: Yes, for false imprisonment, because the bystander locked the man

D: No, because the bystander was privileged to act as he did. D is correct. A person is privileged to use reasonable force to protect another against imminent harm. The privilege includes preventing self-inflicted harm. Here, the intoxicated man was on the verge of harming or killing himself. It was reasonable for the bystander to push the man and lock him in the bedroom until the danger had passed.

A plaintiff suffered a serious injury while participating in an impromptu basketball game at a public park. The injury occurred when the plaintiff and the defendant, on opposing teams, each tried to obtain possession of the ball when it rebounded from the backboard after a missed shot at the basket. During that encounter, the plaintiff was struck and injured by the defendant's elbow. The plaintiff now seeks compensation from the defendant. At the trial, evidence was introduced tending to prove t

D: No, because the defendant did not intentionally use force that exceeded the players' consent D is correct. The plaintiff gave implied consent to this type of contact by the defendant based on the fact that the game had started out rough, the plaintiff had engaged in similar behavior already, and the defendant did not intentionally use force that went beyond that scope of contact.

A landowner who owned a large tract of land in the mountains sought to protect a herd of wild deer that lived on part of the land. Although the landowner had posted signs that said, "No Hunting—No Trespassing," hunters frequently intruded to kill the deer. Recently, the landowner built an eight-foot-high chain-link fence, topped by three strands of barbed wire, across a gully on her land that provided the only access to the area where the deer lived. A wildlife photographer asked the lando

D: No, because the potential for harm created by the presence of the barbed wire was apparent. D is correct. The landowner employed reasonable use of force (the barbed wire) to defend her property, without any attempt to conceal it, which was apparent to the photographer. An average person would stay away from protruding, sharp barbed wire covering the top of a fence. However, the photographer disregarded the obvious risk and proceeded anyway, allowing himself to become severely injured.

A landlord owns and operates a 12-story apartment building containing 72 apartments, 70 of which are rented. A pedestrian has brought an action against the landlord alleging that while he was walking along a public sidewalk adjacent to the landlord's apartment building a flower pot fell from above and struck him on the shoulder, causing extensive injuries. The action was to recover damages for those injuries. If the pedestrian proves the foregoing facts and offers no other evidence explaining th

D: No, because there is no basis for a reasonable inference that the landlord was negligent. D is correct. Any time negligence must be inferred, there is a res ipsa loquitur issue. For a claim based on res ipsa loquitur to prevail, the pedestrian must show that the landlord had exclusive control of the flowerpot before it fell. In this case, the landlord did not have exclusive control of the flowerpot during the relevant time frame because 70 of the units had tenants, so res ipsa loquitur may not be used to establish negligence. Therefore, the pedestrian will not be able to make his prima facie case, and the landlord's motion for a directed verdict should be granted.

A sporting goods shop was burglarized by an escaped inmate from a nearby prison. The inmate stole a rifle and bullets from a locked cabinet. The burglar alarm at the shop did not go off because the shop's owner had negligently forgotten to activate the alarm's motion detector. Shortly thereafter, the inmate used the rifle ammunition stolen from the shop in a shooting spree that caused injury to several people, including the plaintiff. If the plaintiff sues the shop's owner for the injury she suf

D: No, because there is no evidence of circumstances suggesting a high risk of theft and criminal use of firearms stocked by the shop's owner. D is correct. The commission of a criminal act often supersedes the liability of the original negligent actor. An exception occurs if the negligent act creates a condition such that a criminal act is the foreseeable consequence of that action. Here, the shop owner didn't forget to lock his door; he just forgot to set the alarm on his gun shop. The negligence of not turning on the alarm does not contribute to the likelihood of the crime happening. In this case, the inmate broke in not knowing if the alarm was on or off. The break-in was a superseding force that would cut off the defendant's liability. It is not foreseeable that the breaking would happen just because there is a prison nearby unless it is a notably high crime area already.

As a shopper was leaving a supermarket, an automatic door that should have opened outward opened inward, striking and breaking the shopper's nose. The owner of the building had installed the automatic door. The lease, pursuant to which the supermarket occupied the building, provided that the supermarket was responsible for all maintenance of the premises. The shopper sued the supermarket. At trial, neither the shopper nor the supermarket offered any testimony, expert or otherwise, as to why th

D: Submit the case to the jury, because on these facts negligence may be inferred. D is correct. Based on the evidence in the record, the jury could infer negligence because the accident occurred from a door in the exclusive control of the supermarket, and it is not the type of accident that would occur without negligence. Either way, this is question for a jury to decide.

A wealthy elderly woman was repeatedly harassed by a debt collector over a period of two months. The debt collector was trying to collect a large debt owed to his client by the woman's impoverished adult son. Although the debt collector knew that the woman was not legally responsible for the son's debt, he called the woman multiple times each day and threatened to destroy her credit. He also told her that he knew where she lived and that he was going to withdraw the money from her bank accou

D: The debt collector's conduct was extreme and outrageous D is correct. A conclusion that the debt collector's conduct was extreme and outrageous, combined with the fact that the woman experienced severe emotional distress and that the debt collector was probably at least reckless with respect to the risk of causing her distress, will provide the woman with a plausible claim for intentional infliction of emotional distress (IIED). None of the conclusions stated in the other answer choices provides as much support to the woman's IIED claim.

An ordinance in a small town required all restaurants to designate smoking and nonsmoking sections for their customers. A cigarette smoker and a nonsmoker were seated at adjoining tables in a small restaurant. The smoker's table was in the smoking section, and the nonsmoker's table was in the nonsmoking section. When the smoker lit a cigarette, the nonsmoker politely requested that he not smoke, explaining that she had a severe allergy to cigarette smoke. The smoker ignored the nonsmoker's

D: Was the smoker's conduct unreasonable under the circumstances? D is correct. Whether the smoker's conduct was reasonable is irrelevant. The defendant's intent to cause harmful or offensive contact with the plaintiff is what would sustain a battery claim. Reasonableness would be relevant to determining negligence, not whether a battery occurred.

In preparation for a mountain-climbing expedition, a climber purchased the necessary climbing equipment from a retail dealer in sporting goods. A week later, the climber fell from a rock face when a safety device he had purchased from the retail dealer malfunctioned because of a defect in its manufacture. Thereafter, a rescuer was severely injured when he tried to reach and give assistance to the climber on the ledge to which the climber had fallen. The rescuer's injury was not caused by any fau

D: Yes, because injury to a person in the rescuer's position was foreseeable if the safety device failed. D is correct. This is a products liability question. The rescuer can succeed in the claim against the retailer because injury to a person in the rescuer's position was foreseeable if the safety device failed.

A rancher and his neighbor were involved in a boundary dispute. In order to resolve their differences, each drove his truck to an open pasture area on his land where the two properties were separated by a fence. The rancher was accompanied by four friends, and the neighbor was alone. The neighbor got out of his truck and walked toward the fence. The rancher got out but simply stood by his truck. When the neighbor came over the fence, the rancher shot him, inflicting serious injury. In a battery

D: Yes, because it was unreasonable for the rancher to consider the use of a gun necessary for self-defense. D is correct. The rancher will be liable for battery because his use of force sufficient to cause serious bodily injury was not supported by a reasonable belief that the neighbor was armed or that the neighbor even intended to harm him. Absent such a showing, the rancher will not be privileged in his shooting of the neighbor.

A boater, caught in a sudden storm and reasonably fearing that her boat would capsize, drove the boat up to a pier, exited the boat, and tied the boat to the pier. The pier was clearly marked with "NO TRESPASSING" signs. The owner of the pier ran up to the boater and told her that the boat could not remain tied to the pier. The boater offered to pay the owner for the use of the pier. Regardless, over the boater's protest, the owner untied the boat and pushed it away from the pier. The boat was l

D: Yes, because the boater was privileged to enter the owner's property to save her boat. D is correct. The boater was privileged to trespass on the owner's property under the doctrine of private necessity, because the boater's property was at risk. Because the boater's intrusion onto the pier was privileged, the owner had no right to exclude her or her boat from the pier. When the owner untied the boat, he committed an unprivileged trespass upon the boater's property, so the owner must pay for the loss of the boat.

When a tire of a motorist's car suffered a blowout, the car rolled over and the motorist was badly injured. Vehicles made by the manufacturer of the motorist's car have been found to be negligently designed, making them dangerously prone to rolling over when they suffer blowouts. A truck driver who was driving behind the motorist when the accident occurred stopped to help. Rescue vehicles promptly arrived, and the truck driver walked along the side of the road to return to his truck. As he a

D: Yes, because the car manufacturer's negligence caused the dangerous situation that invited the rescue by the truck driver. D is correct. By negligently designing the vehicles, the manufacturer became liable for foreseeable intervening causes of harm, including the invitation of rescue. There is no evidence that the truck driver was negligent or acted recklessly in assisting the motorist, and he will thus prevail.

An eight-year-old child went to the grocery store with her mother. The child pushed the grocery cart while her mother put items into it. The child's mother remained near the child at all times. Another customer in the store noticed the child pushing the cart in a manner that caused the customer no concern. A short time later, the cart the child was pushing struck the customer in the knee, inflicting serious injury. Assume that the child was negligent and the child's mother did not adequately sup

D: Yes, because the child's mother did not adequately supervise the child's actions. D is correct. Parents have a number of affirmative duties based on their special relationship to their minor children. This includes the duty to exercise reasonable care in the control of the parent's minor children. A parent who is physically present and fails to exercise control of her child is generally not vicariously liable for the child's tortious behavior; rather, the parent may be liable for her own negligence in failing to control the child. Because the child's mother was not adequately supervising her daughter, and it was foreseeable that the child could potentially injure someone, the customer is likely to prevail.

A plaintiff entered a drug store to make some purchases. As he was searching the aisles for various items, he noticed a display card containing automatic pencils. The display card was on a high shelf behind a cashier's counter. The plaintiff saw a sign on the counter that read, "No Admittance, Employees Only." Seeing no clerks in the vicinity to help him, the plaintiff went behind the counter to get a pencil. A clerk then appeared behind the counter and asked whether she could help him. He said

D: Yes, because the clerk had reason to believe that the plaintiff was unaware of the open shaft. D is correct. As an invitee, and even as a trespasser once he went into the area of the store he was not permitted in, the plaintiff was owed a duty by the store, and by extension the clerk, to be warned of known, non-obvious, artificial dangers and for the premise to be made reasonably safe. The fact that the clerk knew the open shaft was there and that the plaintiff likely did not know, will render the store liable for injuries the plaintiff suffered, even if he became a trespasser in ignoring the sign.

A 13-year-old girl was operating a high-speed motorboat. The boat was towing a 9-year-old boy in an inner tube tied to the rear of the motorboat by a rope. The rope became tangled around the boy's foot, causing him to suffer severe injuries. In a suit brought on the boy's behalf against the girl, the boy has introduced uncontroverted evidence that the girl drove carelessly in such a way as to entangle the boy in the rope. Is the boy likely to prevail? A: No, because the boy assumed the ris

D: Yes, because the girl will be held to an adult standard of care. D is correct. In order to determine if the boy is likely to prevail, we must first determine what duty of care was owed to him. The girl is only 13 years old. A majority of courts will hold that a child is required to conform to the standard of care a child of like age, education, intelligence, and experience. However, when a child engages in an activity that is normally one that only adults engage in, courts will hold that the child will be required to conform to the same standard of care as an adult in such an activity. Here, the girl was operating a high-speed motorboat: an adult activity. Therefore, she will be held to an adult standard of care and the boy will prevail.

A shopper slipped and fell in a grocery store, injuring her wrist. In a medical malpractice action against the doctor who treated her, the shopper alleges that the doctor worsened the injury by his treatment. Normally, competent medical treatment would have resulted in a complete cure of the wrist injury. The doctor is seeking to implead the grocery store. The grocery store contends that its alleged negligence was not a proximate cause of any of the injuries allegedly caused by the doctor. Shoul

D: Yes, because the fact-finder could assign some of the responsibility for the shopper's injuries to the grocery store. D is correct. The doctor may implead the grocery store because the fact-finder could conclude that the negligence of both the grocery store and the doctor contributed to the total injury suffered by the shopper, including the worsening of her initial injury that resulted from the doctor's malpractice.

A farmer kept antiques in an uninhabited farmhouse on his property. The farmhouse had been broken into several times in the past, and some of the farmer's goods had been stolen. Instead of posting "No Trespassing" signs, the farmer decided to install an alarm system to deter intruders. While the farmer was in the farmhouse installing the alarm system, he heard a window open in the adjoining room. The farmer crept very quietly to the door of the room, threw the door open, and found an intru

D: Yes, because the farmer used excessive force. D is correct. The farmer was privileged to use reasonable force to prevent or end a trespasser's intrusion upon his land or to protect his property, but he was not privileged to use force that threatened serious bodily injury unless he was himself in danger of serious bodily harm. The force the farmer used was sufficient to and did in fact cause serious bodily injury. The child appeared to pose no threat of bodily harm to the farmer and could have been deterred by less forceful means.

A trucker driving down an isolated country road late one night struck cattle that had escaped from a farmer's pen and wandered into the road. The trucker was unable to stop before hitting the cattle but was not driving carelessly. While he was not injured in the collision, the trucker sustained damage to his truck and lost income during the time it took to repair the truck. The trucker sued the farmer for his damages and invoked the doctrine of res ipsa loquitur. At trial, the farmer introduce

D: Yes, because the jury could conclude that cattle would not ordinarily escape a strong, secure cattle pen in the absence of negligence. D is correct. The facts indicate that the farmer was in exclusive control of his farm and that the trucker did not contribute to the accident. Therefore, the jury is permitted to conclude that the cattle escaped due to the farmer's negligence.

A man's car sustained moderate damage in a collision with a car driven by a woman. The accident was caused solely by the woman's negligence. The man's car was still drivable after the accident. Examining the car the next morning, the man could see that a rear fender had to be replaced. He also noticed that gasoline had dripped onto the garage floor. The collision had caused a small leak in the gasoline tank. The man then took the car to a mechanic, who owns and operates a body shop, and arranged

D: Yes, because the mechanic's injury was a proximate consequence of the woman's negligent driving. D is correct. It is foreseeable that a car accident could rupture a gas tank leading the gasoline to ignite and causing severe burn damage to anyone in or near the car, and the woman's negligence was the proximate cause of the accident and all foreseeable injuries. The manner of the accident is not determinative; the mechanic's injury was a foreseeable result of the accident. The mechanic's injury was not so removed in time and circumstance as to offend fundamental fairness.

A pedestrian was injured when hit by a chair that was thrown from an upper-story hotel window. The pedestrian sued the occupants of all the rooms from which the chair might have been thrown. At trial, the pedestrian has been unable to offer any evidence as to the exact room from which the chair was thrown. The defendants have filed a motion for a directed verdict. Should the court grant the motion? A: No, because it is unreasonable to expect the pedestrian to prove which of the defendants caus

D: Yes, because the pedestrian has failed to offer evidence that the defendants jointly engaged in tortious conduct. D is correct. When two or more tortious acts combine to cause an indivisible injury to a plaintiff, each tortfeasor is jointly and severally liable for that injury. Defendants in a civil suit can be held jointly and severally liable only if their concurrent acts brought about the harm to the plaintiff. These acts do not have to be simultaneous, they only must contribute to the same event. Here, only one occupant threw a chair out of the upper-story hotel window. There is no evidence that the defendants all contributed to the harm that the pedestrian suffered. Therefore, the motion for directed verdict should be granted.

A fire that started in the defendant's warehouse spread to the plaintiff's adjacent warehouse. The defendant did not intentionally start the fire, and the plaintiff can produce no evidence as to how the fire started. However, the defendant had failed to install a sprinkler system, which was required by a criminal statute. The plaintiff can produce evidence that had the sprinkler system been installed, it could have extinguished the fire before it spread. In an action by the plaintiff against

D: Yes, because the plaintiff was harmed as a result of the defendant's violation of a statute that was meant to protect against this type of occurrence. D is correct. A criminal statute can be used to set the standard of care in a negligence action if it was intended to protect against the type of harm that occurred by specifying preventive steps that should be taken. In that case, violation of the statute is negligence per se.

The plaintiff is being treated by a physician for asbestosis, an abnormal chest condition that was caused by his on-thejob handling of materials containing asbestos. His physician has told him that the asbestosis is not presently cancerous, but that it considerably increases the risk that he will ultimately develop lung cancer. The plaintiff brought an action for damages, based on strict product liability, against the supplier of the materials that contained asbestos. The court in this jurisdict

D: Yes, because the plaintiff's emotional distress arises from bodily harm caused by his exposure to asbestos. This is an instance where a defendant (the supplier) is liable for a physical injury (the chest condition and related prognosis) and the plaintiff's emotional distress stems from that underlying tortious conduct. Because the plaintiff may recover for all consequences flowing from this underlying liability, he may tack on the emotional distress damages.

A mother purchased an expensive television from an appliance store for her adult son. Two years after the purchase, a fire started in the son's living room in the middle of the night. The fire department concluded that the fire had started in the television. No other facts are known. The son sued the appliance store for negligence. The store has moved for summary judgment. Should the court grant the store's motion? A: No, because televisions do not catch fire in the absence of negligence. B:

D: Yes, because there is no evidence of negligence on the part of the store. D is correct. The son is suing in negligence, not in strict liability. To make out a prima facie case in negligence, the son must introduce evidence that the store was negligent. However, the son has not pointed to any negligent action or omission by the store. This is not an appropriate case for res ipsa loquitur, because the manufacturer, rather than the store, may have been negligent or the negligence may have occurred after the sale (for example, during a repair or while the television was being used by the son).

A law student rented a furnished apartment. His landlord began to solicit his advice about her legal affairs, but he refused to provide it. The landlord then demanded that he vacate the apartment immediately. The landlord also began engaging in a pattern of harassment, calling the student at home every evening and entering his apartment without his consent during times when he was at school. During these unauthorized visits, she removed the handles from the bathroom and kitchen faucets, making t

D: Yes, for injunctive relief, compensatory damages, and punitive damages. D is correct. There is evidence supporting compensatory damages (for emotional distress and the removal of the faucet handles) and punitive damages (based on the landlord's malicious intent and ill will). Because the lease is still in effect and the trespasses are repeated and ongoing, injunctive relief should also be available.

While a plaintiff was leaving an elevator, it suddenly dropped several inches, causing her to fall. An investigation of the accident revealed that the elevator dropped because it had been negligently maintained by an elevator company. The elevator company had a contract with the owner of the building to inspect and maintain the elevator. The plaintiff's fall severely aggravated a preexisting physical disability. If the plaintiff sues the elevator company for damages for her injuries, she should

D: damages for the injury caused by the falling elevator, including the aggravation of her preexisting disability. D is correct. It was foreseeable by the elevator company that a failure to maintain an elevator would cause severe physical harm to its passengers should the elevator malfunction as the result of that negligence. Clearly, the burden to maintain the elevator, which the elevator company was being paid for, did not outweigh the potential harm to the elevator passengers. When negligence is found, the defendant takes the plaintiff as it finds her. The elevator company is therefore responsible for the plaintiff's injuries, including the aggravation of her existing disability.

In an action brought against a defendant by a pedestrian's legal representative, the only proof that the legal representative offered on liability were that: (1) the pedestrian was killed instantly while walking on the shoulder of the highway; (2) the defendant was driving the car that struck the pedestrian; and (3) there were no living witnesses to the accident other than the defendant, who denied negligence. The jurisdiction has adopted a rule of pure comparative negligence. If, at the end of

D: deny the motion, because, in the circumstances, negligence on the part of the defendant may be inferred. D is correct. The pedestrian's representative introduced sufficient evidence for a res ipsa loquitur claim, which has the effect of allowing the jury to decide whether to infer the defendant's negligence. A directed verdict for the defendant would thus be improper because the jury must decide whether it will infer the defendant's negligence or find that his denial of negligence overcomes the res ipsa showing


Related study sets

intro to nursing - culture (week 4 ch 9)

View Set

Chapter 1 Lesson 3 Health risks and behavior

View Set

Module 2: Engagement Planning, Obtaining an Understanding of the Client and Assessing Risks

View Set

BIOL252 07: Chpt 10 appendicular skeleton

View Set